ChaseDream

标题: GWD-6-20以前没人问过 [打印本页]

作者: blackhorse    时间: 2004-8-17 22:30
标题: GWD-6-20以前没人问过

Q20:

Five years ago, as part of a plan to encourage citizens of Levaska to increase the amount of money they put into savings, Levaska’s government introduced special savings accounts in which up to $3,000 a year can be saved with no tax due on the interest unless money is withdrawn before the account holder reaches the age of sixty-five.        Millions of dollars have accumulated in the special accounts, so the government’s plan is obviously working.

Which of the following, if true, most seriously weakens the argument?

  1. A substantial number of Levaskans have withdrawn at least some of the money they had invested in the special accounts.
  2. Workers in Levaska who already save money in long-term tax-free accounts that are offered through their workplace cannot take advantage of the special savings accounts introduced by the government.
  3. The rate at which interest earned on money deposited in regular savings accounts is taxed depends on the income bracket of the account holder.
  4. Many Levaskans who already had long-term savings have steadily been transferring those savings into the special accounts.
  5. Many of the economists who now claim that the government’s plan has been successful criticized it when it was introduced.

答案:D。有点不明白。感觉D是支持呀。为什么不选A?



作者: mindfree    时间: 2004-8-17 23:53

You need to know what the conclusion is and what the premise is.

The conclusion is that the account encourage saving. However, if D is true and the money in that account is just savings from other account, the special account did not have the expected result.

As to A, you need to ask yourself, if A is correct, does the conclusion hold, i.e. does the account encourage saving? I say yes because withdrawn and saving are different. The fact that there is withdraw is out of scope for the conclusion.  


作者: blackhorse    时间: 2004-8-18 18:04
谢谢总教头,我再好好领会一下。
作者: blackhorse    时间: 2004-8-18 18:09
做这道题目的时候感觉D就是一个典型的逻辑答案,但就是没有想明白为什么。原来我把结论想错了。
作者: tai6996    时间: 2004-10-18 23:21
總教頭果然就是總教頭....太厲害了...
作者: yuliyang    时间: 2005-4-2 17:41
佩服!!!
作者: Dennies    时间: 2005-4-13 13:29

看来大家对这道题的答案都很统一了,想说说我自己的一点想法,开始时我也选的A,根本就没有重视D,D说甚么都么看完.

我认为A有加强的作用,既然有相当的人撤回了存款,但仍然达到了Millions of dollars have accumulated in the special accounts的效果,更说明了方案的有效性支持了结论.大家觉的呢?


作者: crossrose    时间: 2005-4-13 16:56

d is right. Gut


作者: foucsfei    时间: 2005-6-14 23:50

D 在吸引存款上是成功的。是支持了结论,而非削若。


但是原文是要求


Which of the following, if true, most seriously weakens the argument?


Please help


作者: catcenter    时间: 2006-5-5 18:01

这道题还表明像E这样的,诉诸于权威的选项都是干扰选项,可以不与考虑



作者: bloodymica    时间: 2006-7-29 16:49
以下是引用blackhorse在2004-8-17 22:30:00的发言:

Q20:

Five years ago, as part of a plan to encourage citizens of Levaska to increase the amount of money they put into savings, Levaska’s government introduced special savings accounts in which up to $3,000 a year can be saved with no tax due on the interest unless money is withdrawn before the account holder reaches the age of sixty-five.  Millions of dollars have accumulated in the special accounts, so the government’s plan is obviously working.

Which of the following, if true, most seriously weakens the argument?

  1. A substantial number of Levaskans have withdrawn at least some of the money they had invested in the special accounts.
  2. Workers in Levaska who already save money in long-term tax-free accounts that are offered through their workplace cannot take advantage of the special savings accounts introduced by the government.
  3. The rate at which interest earned on money deposited in regular savings accounts is taxed depends on the income bracket of the account holder.
  4. Many Levaskans who already had long-term savings have steadily been transferring those savings into the special accounts.
  5. Many of the economists who now claim that the government’s plan has been successful criticized it when it was introduced.

答案:D。有点不明白。感觉D是支持呀。为什么不选A?


题目明白了,可是从来就没看懂C到底是什么意思……谁能帮我解释一下~~


作者: 挥着猪翅膀    时间: 2006-7-29 22:54

actually, D gives a possible alternative factor that undermine the argument.

Besides, it's simple for u to get the right answer by excluding those irrelevant choices.


作者: zhaoyak7    时间: 2006-7-30 00:06

D没有问题,但偶再较一下真儿:题干说存够了才行,而很多人不停地从这个帐户中拿钱,不也削弱了结论吗,即"这个帐户"对增加储蓄没有作用?

A substantial number of Levaskans have withdrawn at least some of the money they had invested in the special accounts.


作者: stexms    时间: 2007-9-20 14:59

the object of the plan is to increase the amount of maney the citizens of Levaska deposit into the bank account, so wether the plan is working is based on wether the deposit has increased.


作者: shirley8707    时间: 2008-5-8 15:24

作者: ppjelly    时间: 2008-5-10 23:12

    

Five years ago, as part of a plan to
encourage citizens of Levaska to increase the amount of money they put into
savings, Levaska’s government introduced special savings accounts in which up
to $3,000 a year can be
saved with no tax due on the interest unless money is withdrawn before the
account holder reaches the age of sixty-five
. 
Millions of dollars have accumulated in the special accounts, so the
government’s plan is obviously working.
Which of the following, if true, most
seriously weakens the argument?
A.A substantial number of Levaskans have withdrawn at least some
     of the money they had invested in the special accounts.

偶也错选了A,在仔细分析一下,估计选A的人是像我这么想的,存钱并且保持一段时间才是银行赢利的充分必要条件。政府要求超过$3000在65岁之前不要取出,如果我们认为这是此项计划成功的一个必要条件,那么A说取出了是不是就破坏了这个条件呢?其实不是,请注意看A说有相当一部分人取了一些钱出来,可是没说这些人没到65岁啊!所以即使有人取钱了,也不一定破坏了政府的要求“65岁之前不能取出”。所以比较一下D更万无一失的正确,所以选D:)



作者: meixu11    时间: 2009-6-6 18:00

解答相当清晰~    谢谢斑竹~   :)

也不知道斑竹现在会不会常回来看看呀~    :)


作者: zhengjingzhe    时间: 2009-7-20 09:05
up
作者: Journal    时间: 2009-7-24 22:56
以下是引用ppjelly在2008/5/10 23:12:00的发言:

 

Five years ago, as part of a plan to
encourage citizens of Levaska to increase the amount of money they put into
savings, Levaska’s government introduced special savings accounts in which up
to $3,000 a year can be
saved with no tax due on the interest unless money is withdrawn before the
account holder reaches the age of sixty-five
. 
Millions of dollars have accumulated in the special accounts, so the
government’s plan is obviously working.
Which of the following, if true, most
seriously weakens the argument?
A.A substantial number of Levaskans have withdrawn at least some
     of the money they had invested in the special accounts.

偶也错选了A,在仔细分析一下,估计选A的人是像我这么想的,存钱并且保持一段时间才是银行赢利的充分必要条件。政府要求超过$3000在65岁之前不要取出,如果我们认为这是此项计划成功的一个必要条件,那么A说取出了是不是就破坏了这个条件呢?其实不是,请注意看A说有相当一部分人取了一些钱出来,可是没说这些人没到65岁啊!所以即使有人取钱了,也不一定破坏了政府的要求“65岁之前不能取出”。所以比较一下D更万无一失的正确,所以选D:)


先谢谢! 可是我在另外一个帖子里面说D里面的Many  L. who already... 的many 不足为据因为要的是weaken而不是deny。也就是我们就算假设只有10个人也是一种weaken,我可以理解。

如果同样放到A来说就算只有10个人把钱放进去又拿出来,这并没有达到increase the saving amt的效果啊?也算是一种weaken 他们有没有超过65岁,没说超过65,也没说少于65. . 我还是没有办法理解


作者: sherry树丛    时间: 2009-11-13 15:15
我觉得A错,是因为即使大部分的人把原来存的钱又给取了出来,但我们不能排除一种情况就是另外一部分人所投的钱要比所取出的钱多的多,如果按照这种情况发展的话,那么A就达不倒weaken 的效果了。
请指正。
作者: sherry树丛    时间: 2009-11-13 15:33
刚刚又想了一下,好像我上面的回答有些漏洞,呵呵。现在我觉得是withdrawn与savings是两个不同的概念,虽然有很多人把钱撤了出来,但是存款总额还是不断的在增长,所以可以见,投入该项目的资金要比撤出的数额大的多,从而说明政府的这项计划起到了效果,所以A为支持。至于D,确实是weaken,因为它说明了存款额的大量增长是因为人们把钱从长期存款中转移到special account中的,而非是吸引新的存款。所以该项计划失败。




欢迎光临 ChaseDream (https://forum.chasedream.com/) Powered by Discuz! X3.3